Difference between revisions of "2021 Fall AMC 10A Problems/Problem 16"

m (Problem)
Line 1: Line 1:
 
==Problem==
 
==Problem==
The graph of <math>f(x) = |\lfloor x \rfloor| - |\lfloor 1 - x \rfloor|</math> is symmetric about which of the following? (Here <math>\lfloor x \rfloor</math> is the greatest integer not exceeding <math>x</math>.)
+
The graph of <cmath>f(x) = |\lfloor x \rfloor| - |\lfloor 1 - x \rfloor|</cmath> is symmetric about which of the following? (Here <math>\lfloor x \rfloor</math> is the greatest integer not exceeding <math>x</math>.)
  
 
<math>\textbf{(A) }</math> the <math>y</math>-axis <math>\qquad \textbf{(B) }</math> the line <math>x = 1</math> <math>\qquad \textbf{(C) }</math> the origin <math>\qquad
 
<math>\textbf{(A) }</math> the <math>y</math>-axis <math>\qquad \textbf{(B) }</math> the line <math>x = 1</math> <math>\qquad \textbf{(C) }</math> the origin <math>\qquad

Revision as of 23:35, 24 November 2021

Problem

The graph of \[f(x) = |\lfloor x \rfloor| - |\lfloor 1 - x \rfloor|\] is symmetric about which of the following? (Here $\lfloor x \rfloor$ is the greatest integer not exceeding $x$.)

$\textbf{(A) }$ the $y$-axis $\qquad \textbf{(B) }$ the line $x = 1$ $\qquad \textbf{(C) }$ the origin $\qquad \textbf{(D) }$ the point $\left(\dfrac12, 0\right)$ $\qquad \textbf{(E) }$ the point $(1,0)$

Solution

IN PROGRESS AND WILL FINISH SOON. NO EDIT PLEASE. A MILLION THANKS.

~MRENTHUSIASM

See Also

2021 Fall AMC 10A (ProblemsAnswer KeyResources)
Preceded by
Problem 15
Followed by
Problem 17
1 2 3 4 5 6 7 8 9 10 11 12 13 14 15 16 17 18 19 20 21 22 23 24 25
All AMC 10 Problems and Solutions

The problems on this page are copyrighted by the Mathematical Association of America's American Mathematics Competitions. AMC logo.png